3
$\begingroup$

Let $u^i_j$, $i,j = 1, . . . N$, and det$_q^{-1}$ be the standard generators of the quantum group $U_q(N,C)$, and define the matrices $U$ and $U^{\ast}$ by setting $U_{ij} := u^i_j$ and $U^{\ast}_{ij}:=(u^j_i)^{\ast}$. It is "well known" that $U^{\ast}U=UU^{\ast}=1$. How does one prove this?

Moreover, how does this imply that $u^i_j(u^r_s)^{\ast} = (u^r_s)^{\ast}u^i_j$, for $r\neq i,s\neq j$?

$\endgroup$
2
  • $\begingroup$ Is something missing after "for"? $\endgroup$ Nov 6, 2010 at 22:56
  • $\begingroup$ It's fixed now. $\endgroup$ Nov 7, 2010 at 1:50

1 Answer 1

2
$\begingroup$

I assume that $U^\ast_{ij}=S(u_j^i)$, where $S$ is the antipode. (See for example the book by Klimyk & Schmüdgen, Section 9.2.4.) If so, then $UU^\ast=1=U^\ast U$ follows from the antipode axiom

$\mu\circ(\mathrm{Id}\otimes S)\circ \Delta =\varepsilon =\mu\circ( S\otimes\mathrm{Id})\circ \Delta$

after applying both sides to an arbitrary generator $u_j^i$.

To prove $u_j^iS(u_s^r)=S(u_s^r)u_j^i$ for $r\neq i, s\neq j$, one can multiply both sides by the quantum determinant and use that it is central to get the equivalent statement $u_j^iM_s^r=M_s^r u_j^i$, where $M_s^r$ are quantum minors of size $(N-1)\times(N-1)$. Now one can observe that the subalgebra generated by $u_b^a$ with $a\neq r, b\neq s$ is isomorphic to $M_q(N-1,C)$, the quantized algebra of regular functions on all $(N-1)\times(N-1)$ complex matrices. $u_j^i$ and $M_s^r$ belong to this subalgebra and moreover $M_s^r$ is the quantum determinant in $M_q(N-1,C)$, hence it commutes with $u_j^i$.

$\endgroup$
3
  • $\begingroup$ Nice answer, but I think there's a slight error. I find in K & S $(u^i_j)^* = S(u^j_i)$, and not, $(u^i_j)^* = S(u^i_j)$. Reworking your proof then gives $u^i_jS(u^r_s) = S(u^r_s)u^i_j$, for $r \neq j,s \neq j$. The result you have stated is not true. No worries though, you still gave me the answer I was looking for. Do you think you could vary your proof to find a formula for $u^i_jS(u^j_s) - S(u^j_s)u^i_j$, which is certainly non-zero? $\endgroup$ Nov 11, 2010 at 20:16
  • $\begingroup$ Glad you found it useful! Actually I think it should be $(U^\ast)_{ij}=(u_i^j)^\ast$ and not $(u_j^i)^\ast$ as you write, because the "hermitian conjugate" of $U$ should be the transpose matrix with all entries "conjugated" i.e. starred. This is also consistent with KS. If I'm not mistaken. If you would like to prove that something is nonzero, sometimes a good way is to find a natural representation where it is nonzero. However, I think one can prove that a factor q enters there, and this is discussed somewhere on mathoverflow. $\endgroup$ Nov 11, 2010 at 23:13
  • $\begingroup$ Check this out: mathoverflow.net/questions/41628/… $\endgroup$ Nov 11, 2010 at 23:20

Your Answer

By clicking “Post Your Answer”, you agree to our terms of service and acknowledge you have read our privacy policy.

Not the answer you're looking for? Browse other questions tagged or ask your own question.